LSAT and Law School Admissions Forum

Get expert LSAT preparation and law school admissions advice from PowerScore Test Preparation.

 Administrator
PowerScore Staff
  • PowerScore Staff
  • Posts: 8916
  • Joined: Feb 02, 2011
|
#35112
Complete Question Explanation

Assumption. The correct answer choice is (E)

Your task in this Assumption question is to select the answer that contains information required for
the conclusion to be valid. Reordered, the argument proceeds:

..... Premise: ..... syndicated political columnists often use their newspaper columns to try to
..... ..... ..... ..... persuade readers to vote a certain way

..... Premise: ..... by the time such a column appears, nearly all who will vote in the election
..... ..... ..... ..... will have already made a decision about which candidate to vote for

..... Conclusion: ..... thus, the political columnists’ efforts to persuade voters rarely succeed

Because there is no “rogue” information in the conclusion that did not appear previously in the
argument, your prephrase begins with the understanding that you are dealing with a Defender style
assumption question. The weakness in the argument is that it assumes that because voters have
already made a decision about voting before the column appears, that their decision cannot be
affected by the column’s attempts to persuade.

The correct answer will likely raise the possibility that a voter’s initial decision cannot be changed,
but will do so in order to dismiss it, thus defending the conclusion against an attack on that
weakness. The incorrect answers will not contain information required for the conclusion to be
invalid. Instead, they may have no effect on the conclusion, support the conclusion while not be
required information, or could undermine the conclusion.

Answer choice (A): This choice is incorrect because it has no effect on the conclusion, which
allowed that the columnists’ efforts would sometimes succeed, and was based on the notion that
nearly all voters have reached a decision about which candidate to vote for before the column
appears.

Answer choice (B): This choice has no effect on the conclusion, which depended on nearly all voters
having already made up their minds, and therefore not being persuaded by the columns at all.

Answer choice (C): As with choice (B), this answer has no effect on the argument, which pertained
only to the question of whether the columns appear too late to have any effect on the readers.

Answer choice (D): This choice has no effect on the conclusion, because the argument had nothing
to do with this difference between regular and infrequent readers.

Answer choice (E): This is the correct answer choice. This choice is correct because it raises the
possibility that the readers could change their mind about which candidate to vote for, but does so
only to dismiss it, defending the conclusion from an attack on that basis. If it were not the case that
people rarely can be persuaded after they have made a decision about which candidate to vote for,
then the conclusion would be invalid.
 Juanq42
  • Posts: 29
  • Joined: Jul 21, 2019
|
#67303
Hello,

Maybe I did not take adequate notes during my lesson, but may someone elaborate on the definition for a "Defender style assumption question?" Are there other styles I should remain aware of?

As for the question, in my prephrase, I had the following (incorrect) understanding...

syndicated political columnists are targeting their readers to be persuaded, but nearly all (readers included) who will vote cannot be persuaded.

I believed that the argument assumed that these particular newspaper readers* are less likely to be persuaded and chose D.

I now notice that the mention of "all who will vote cannot be persuaded" could apply to readers and everyone else ( infrequent readers)... but I am only further confused with this choice having no effect... clarification would be appreciated!

Also - Is there any special consideration for the level of certainty expressed throughout the stimulus - did it have any effect on determining answer E? ("often use their newspaper.." "rarely succeed..." "nearly all.." )
 Adam Tyson
PowerScore Staff
  • PowerScore Staff
  • Posts: 5153
  • Joined: Apr 14, 2011
|
#67306
Thanks for the questions, Juanq42! First, as to Defender Assumptions, they are answer choices that fight off an attack on the argument or that eliminate a potential weakness. The other type we discuss in our approach is the Supporter Assumption, which is one that closes a logical gap in the argument by connecting the premises to something new or "rogue" in the conclusion. Supporters are very much like using the "Mechanistic" approach to Justify the Conclusion questions.

These really aren't two types of Assumptions as much as they are two different lenses through which to attack these questions. Consider this argument:

"Mammals give birth to live young, so giraffes must give birth to live babies."

There is an obvious gap in the argument - where did giraffes come from? That's a new element, and we could take a Supporter approach and prephrase a link between the premises and the conclusion. That link would be "giraffes are mammals."

But some people would take a slightly different approach to that argument and look at it through another lens, that of attacking the weakness in the argument. Such a student might see this argument and think to themselves "but what if giraffes aren't mammals? That would be a real problem here!" To fend off this attack, and fix this problem, we would prephrase a Defender answer. And what would that answer be? "Giraffes are mammals." The exact same answer, but arrived at through two different mental processes, one a "close the gap" (Supporter) approach and the other a "eliminate the problem" (Defender) approach.

So, when we talk about "Defender style Assumption questions", we are really talking about which approach we took to get to the answer. Sometimes one approach seems more obvious and intuitive than the other - in my example above, I think the Supporter approach is more natural since "giraffes" were so obviously new in the conclusion. In this case, we lean towards taking a Defender approach because there is an obvious attack available in the form of "but what if a column could change a voter's mind?" The correct answer eliminates that possibility, and thereby defends the argument from that attack.

The problem with answer D is that a comparison between those who regularly read the columns and those who rarely do isn't relevant to the issue of whether the columns could influence voters. It doesn't matter whether one group is more or less likely to be influenced, and negating this answer does nothing to the argument.

I do think the level of certainty in the stimulus is important, because it means we do not want assumption answers that are too certain, like "nobody can be influenced once they have made up their mind." That would be too strong, and is not required by the argument since the author is only arguing about what rarely or nearly always happens. We didn't have any answer choices here that are that clearly overstated, but you will encounter them, so paying attention to the strength of the language is important.
 Juanq42
  • Posts: 29
  • Joined: Jul 21, 2019
|
#67322
Hi Adam!

Thanks for the response.

I understand the distinction between these two approaches better now. I guess I was anticipating new "rogue" info to be found in the answer choices, when in fact, there was no need for it since nothing new was mentioned in the conclusion. Thanks!

Get the most out of your LSAT Prep Plus subscription.

Analyze and track your performance with our Testing and Analytics Package.